2019 AMC 10A Problems/Problem 22

Revision as of 18:16, 16 February 2019 by Pratimakarpe (talk | contribs) (Solution 2: Commenting out solution 2 as it is incorrect. The probability of a difference of exactly 1/2 is 0, not 1/8th, and the probability of <1/2 and >1/2 is not symmetric, as shown in Solution 1)
The following problem is from both the 2019 AMC 10A #22 and 2019 AMC 12A #20, so both problems redirect to this page.

Problem

Real numbers between 0 and 1, inclusive, are chosen in the following manner. A fair coin is flipped. If it lands heads, then it is flipped again and the chosen number is 0 if the second flip is heads and 1 if the second flip is tails. On the other hand, if the first coin flip is tails, then the number is chosen uniformly at random from the closed interval $[0,1]$. Two random numbers $x$ and $y$ are chosen independently in this manner. What is the probability that $|x-y| > \tfrac{1}{2}$?

$\textbf{(A)} \frac{1}{3} \qquad \textbf{(B)} \frac{7}{16} \qquad \textbf{(C)} \frac{1}{2} \qquad \textbf{(D)} \frac{9}{16} \qquad \textbf{(E)} \frac{2}{3}$

Solution 1

There are several cases depending on what the first coin flip is when determining $x$ and what the first coin flip is when determining $y$.

The four cases are:

  1. $x$ is either $0$ or $1$ and $y$ is either $0$ or $1$
  2. $x$ is either $0$ or $1$ and $y$ is chosen from the interval $[0,1]$
  3. $x$ is is chosen from the interval $[0,1]$ and $y$ is either $0$ or $1$
  4. $x$ is is chosen from the interval $[0,1]$ and $y$ is chosen from the interval $[0,1]$

Each case has a $\frac{1}{4}$ chance of occurring.

For case 1, we need $x$ and $y$ to be different. Therefore, the probability for success in case 1 is $\frac{1}{2}$.

For case 2, if $x$ is 0, we need $y$ to be in the interval $(\frac{1}{2}, 1]$. If $x$ is 1, we need $y$ to be in the interval $[0, \frac{1}{2})$. Regardless of what $x$ is, the probability for success for case 2 is $\frac{1}{2}$.

By symmetry, case 3 has the same success rate as case 2.

For case 4, we must use geometric probability because there are an infinite number of pairs $(x, y)$ that can be selected, whether they satisfy the inequality or not. Graphing $|x-y| > \tfrac{1}{2}$, gives us the following picture where the shaded area is the set of all the points that fulfill the inequality:

[asy] filldraw((0,0)--(0,1)--(1/2,1)--(0,1/2)--cycle,black+linewidth(1)); filldraw((0,0)--(1,0)--(1,1/2)--(1/2,0)--cycle,black+linewidth(1)); draw((0,0)--(0,1)--(1,1)--(1,0)--cycle); [/asy]

The shaded area is $\frac{1}{4}$, which means the probability for success for case 4 is $\frac{1}{4}$.

Adding up the success rates from each case, we get:

$(\frac{1}{4}) \cdot (\frac{1}{2} + \frac{1}{2} + \frac{1}{2} + \frac{1}{4}) = \boxed{\textbf{(B) }\frac{7}{16}}$.


See Also

2019 AMC 10A (ProblemsAnswer KeyResources)
Preceded by
Problem 21
Followed by
Problem 23
1 2 3 4 5 6 7 8 9 10 11 12 13 14 15 16 17 18 19 20 21 22 23 24 25
All AMC 10 Problems and Solutions
2019 AMC 12A (ProblemsAnswer KeyResources)
Preceded by
Problem 19
Followed by
Problem 21
1 2 3 4 5 6 7 8 9 10 11 12 13 14 15 16 17 18 19 20 21 22 23 24 25
All AMC 12 Problems and Solutions

The problems on this page are copyrighted by the Mathematical Association of America's American Mathematics Competitions. AMC logo.png